9
$\begingroup$

Does anyone know of any lower bounds on the number of nonzero digits that appear in powers of 2 when written to base 3? (Other than the easy "If it's more than 8 it has to have at least 3.") I know there's been some stuff done with powers of 2 written to base 3, but I can't seem to find anything that quite answers this question.

(Are there more general such bounds? Powers of a written to base b? (To avoid triviality, suppose that no power of a is a power of b... or are stricter conditions needed?) From what I can tell, it looks like even specific instances of these are hard, so I suppose I should stick to the specific version.)

$\endgroup$
8
  • $\begingroup$ This should be a pretty open problem. I can't find a related problem of can the square be written by two digits only. $\endgroup$ Jul 2, 2010 at 23:46
  • $\begingroup$ arxiv.org/abs/math/0512006 $\endgroup$
    – Junkie
    Jul 3, 2010 at 0:07
  • $\begingroup$ @Junkie: Yes, I saw that. It didn't appear to address what I'm asking, though, or at least not that I noticed.... did I miss something? $\endgroup$ Jul 3, 2010 at 0:10
  • $\begingroup$ "Rather than easy" (at least three nonzero digits) is Levi ben Gerson's mathoverflow.net/questions/29926/…. The problem has to do something with simultaneous (special) rational approximations to $\log 2$ and $\log 3$. $\endgroup$ Jul 3, 2010 at 0:16
  • $\begingroup$ So, the insolvability of $a^m-b^n=1$ for $a,b,n,n\ge2$ (Catalan's equation), except $3^2-2^3=1$, implies "at least 3 nonzero digits" for any $a$ and $b$. $\endgroup$ Jul 3, 2010 at 0:25

1 Answer 1

13
$\begingroup$

A nontrivial lower bound can be found in a paper of Cam Stewart (see http://www.math.uwaterloo.ca/PM_Dept/Homepages/Stewart/Jour_Books/J-reine-ange-Math-1980.pdf). He proves, more generally, for fixed bases a and b for which $\log a/\log b$ is irrational, that the sum of the number of nonzero digits in the base a and base b digits of an integer n exceeds (essentially) $$\log \log n/ \log \log \log n.$$

$\endgroup$
1
  • $\begingroup$ Ooh, thank you! Making use of this might not be easy, but it certainly does answer the question, and in generality, too! $\endgroup$ Jul 3, 2010 at 19:48

Your Answer

By clicking “Post Your Answer”, you agree to our terms of service and acknowledge you have read our privacy policy.

Not the answer you're looking for? Browse other questions tagged or ask your own question.